PT29.S4.Q14 - plant manager: we could greatly reduce

dennisgerrarddennisgerrard Member
edited January 2017 in Logical Reasoning 1644 karma
After watching video, I understand why A is true. Yet, I still confuse about (D). Is it not a flaw?
The link is here https://7sage.com/lsat_explanations/lsat-29-section-4-question-14/

Comments

  • SamiSami Live Member Sage 7Sage Tutor
    10789 karma
    So answer choice "D" is talking about a flaw that goes goes from probability in the premise to certainty in the conclusion. For answer choice "D" to be correct, we would need our stimulus to say something like- it's possible that new process will be expensive and the conclusion to say it will cost much.

    But if you take a look at our evidence in our stimulus, does it say that its only probably true? No. It says "not only is the new equipment expensive to buy and install but the new process also costs more". So its a very definite statement. There is no "its possible that its expensive". The plant manager is giving us facts that it is expensive in these ways.

    I hope this helps you with seeing why answer choice "D" is the wrong answer choice.
  • dennisgerrarddennisgerrard Member
    1644 karma
    @Sami Did't expect a quick response. Thanks for your clarification. I understand much better now. (D) is descriptively wrong on the first step.
Sign In or Register to comment.